Page 1 of 4

Special Problem Marathon

Posted: Tue Jun 01, 2021 7:02 pm
by Mursalin
Let \(G\) be the centroid of a right-angled triangle \(ABC\) with \(\angle BCA = 90^\circ\). Let \(P\) be the point on ray \(AG\) such that \(\angle CPA=\angle CAB\), and let \(Q\) be the point on ray \(BG\) such that \(\angle CQB =\angle ABC\). Prove that the circumcircles of triangles \(AQG\) and \(BPG\) meet at a point on side \(AB\).

Re: Special Problem Marathon

Posted: Tue Jun 01, 2021 8:42 pm
by FuadAlAlam
Mursalin wrote:
Tue Jun 01, 2021 7:02 pm
Let \(G\) be the centroid of a right-angled triangle \(ABC\) with \(\angle BCA = 90^\circ\). Let \(P\) be the point on ray \(AG\) such that \(\angle CPA=\angle CAB\), and let \(Q\) be the point on ray \(BG\) such that \(\angle CQB =\angle ABC\). Prove that the circumcircles of triangles \(AQG\) and \(BPG\) meet at a point on side \(AB\).
Let $M$ be the midpoint of $AB$ and $H$ be the feet of the altitude from $C$ to $AB$. We claim that $H$ is the desired point of intersection.
We have $\measuredangle CPG=\measuredangle CPA=\measuredangle BAC=\measuredangle MAC=\measuredangle ACM=\measuredangle ACG$. Thus, $AC$ is tangent to $(CPG)$ at $C$. Now, $AH \cdot AB=AC^2=AG \cdot AP$ which implies that $H$ lies on $(BPG)$. Similarly, $H$ lies on $(AQG)$, as desired.

Re: Special Problem Marathon

Posted: Tue Jun 01, 2021 8:49 pm
by FuadAlAlam
Problem 2:

There are $N$ boxes labelled $B_1, B_2, \ldots, B_N$ which are filled with balls of $N$ different colours $C_1, C_2, \ldots, C_N$. Further, it is known that for each colour we can partition the boxes into two sets, such that the total number of balls of that colour in both sets is the same.
Prove that one can choose a box $B_i$, such that at most half of the balls in box $B_i$ are of colour $C_i$.

Re: Special Problem Marathon

Posted: Tue Jun 01, 2021 10:13 pm
by SMA_Nahian
$\textbf{Solution (P2):}$

Let $D_i$ and $ M_i$ be the number of ball of color $C_i$ in box $B_i$ and not in box $B_i$ respectively. And $S_i$ be the number of ball in $B_i$.

For the sake of contradiction , lets assume that, $$D_i > \dfrac{1}{2} S_i \ \forall \ i \in \{1,2,3, \dots, N\}$$
$$\Rightarrow 2 \cdot D_i > S_i$$
Now,
$$\sum \limits_{i=1} ^N D_i + \sum \limits_{i=1} ^N M_i = \sum \limits_{i=1} ^N S_i < \sum \limits_{i=1} ^N 2\cdot D_i$$
$$\Rightarrow\sum \limits_{i=1} ^N M_i < \sum \limits_{i=1} ^N D_i$$
So, there will be at least a colour $C_j$ suct that $$M_j < D_j$$
So, for any partition of two sets of the boxes, the number of ball coloured $C_j$ is always bigger in the set containing box $B_j$ than the other set. And this contradicts our problem statement. $\blacksquare$

Re: Special Problem Marathon

Posted: Tue Jun 01, 2021 10:26 pm
by SMA_Nahian
$\textbf{Problem 3:}$

Let $a, b, c, d$ be positive real numbers such that $a + b + c + d = 1$. Prove that
,
$$6(a^3 + b^3 + c^3 + d^3) \geq (a^2 + b^2 + c^2 + d^2) + \dfrac{1}{8}$$

Re: Special Problem Marathon

Posted: Tue Jun 01, 2021 11:12 pm
by FuadAlAlam
Solution (P3):

By AM-GM, we have $$4(a^3+b^3+c^3+d^3) \geq (a^3+b^3+c^3+d^3)+(ab^2+bc^2+cd^2+da^2)+(ac^2+a^2c)+(bd^2+b^2d)+(ad^2+ba^2+cb^2+dc^2) \geq (a^2+b^2+c^2+d^2)(a+b+c+d)$$ and $$16(a^3+b^3+c^3+d^3) \geq (a^3+b^3+c^3+d^3)+3(ab^2+bc^2+cd^2+da^2)+3(ac^2+a^2c)+3(bd^2+b^2d)+3(ad^2+ba^2+cb^2+dc^2)+6(abc+bcd+cda+dab) \geq (a+b+c+d)^3$$

Therefore, $6(a^3+b^3+c^3+d^3) = 4(a^3+b^3+c^3+d^3)+2(a^3+b^3+c^3+d^3) \geq (a^2+b^2+c^2+d^2)(a+b+c+d)+ \frac{1}{8}(a+b+c+d)^3 = (a^2+b^2+c^2+d^2)+\frac{1}{8}$ and we are done.

Re: Special Problem Marathon

Posted: Tue Jun 01, 2021 11:16 pm
by FuadAlAlam
Problem 4:

Find all functions $f : \mathbb{Z} \rightarrow \mathbb{Z}$ such that
$$f(x + y) + f(x)^2f(y) = f(y)^3 + f(x + y)f(x)^2$$
for all $x, y \in \mathbb{Z}$.

Re: Special Problem Marathon

Posted: Wed Jun 02, 2021 12:07 pm
by Mehrab4226
FuadAlAlam wrote:
Tue Jun 01, 2021 11:16 pm
Problem 4:

Find all functions $f : \mathbb{Z} \rightarrow \mathbb{Z}$ such that
$$f(x + y) + f(x)^2f(y) = f(y)^3 + f(x + y)f(x)^2$$
for all $x, y \in \mathbb{Z}$.
Let,
$p(x,y)=f(x + y) + f(x)^2f(y) = f(y)^3 + f(x + y)f(x)^2$
Now, by $p(0,0)$ we get,
$f(0)+f(0)^3=f(0)^3+f(0)^3$
Or, $f(0)^3-f(0)=0$
Either, $f(0)=0$, or $f(0)=1$ or, $f(0)=-1$
Now, if $f(0)=0$ then by $p(x,0)$ we get,
$f(x)=f(x)^3$
$\rightarrow f(x)=0,1,-1$ [But the latter two are not acceptable]

Again if $f(0)-1$ then by $p(x,0)$ we get,
$f(x)+f(x)^2=1+f(x)^3$
Solving for $f(x)$ we get,
$f(x)=1,-1$ but $f(x)=-1$ is not acceptable.
Similarly for $f(0)=-1$,
$f(x)=-1,1$ [where the 2nd is not acceptable]

$\therefore f(x)=-1,0,1 $

Re: Special Problem Marathon

Posted: Wed Jun 02, 2021 12:09 pm
by Mehrab4226
Problem: 05
Let $ABC$ be a triangle with circumcentre $O$. The points $D,E,F$ lie in the interiors of the sides $BC,CA,AB$ respectively, such that $DE$ is perpendicular to $CO$ and $DF$ is perpendicular to $BO$. (By interior we mean, for example, that the point $D$ lies on the line $BC$ and $D$ is between $B$ and $C$ on that line.)
Let $K$ be the circumcentre of triangle $AFE$. Prove that the lines $DK$ and $BC$ are perpendicular.

Source:
EGMO 2012(1)

Re: Special Problem Marathon

Posted: Wed Jun 02, 2021 12:33 pm
by Anindya Biswas
Mehrab4226 wrote:
Wed Jun 02, 2021 12:07 pm
FuadAlAlam wrote:
Tue Jun 01, 2021 11:16 pm
Problem 4:

Find all functions $f : \mathbb{Z} \rightarrow \mathbb{Z}$ such that
$$f(x + y) + f(x)^2f(y) = f(y)^3 + f(x + y)f(x)^2$$
for all $x, y \in \mathbb{Z}$.
Let,
$p(x,y)=f(x + y) + f(x)^2f(y) = f(y)^3 + f(x + y)f(x)^2$
Now, by $p(0,0)$ we get,
$f(0)+f(0)^3=f(0)^3+f(0)^3$
Or, $f(0)^3-f(0)=0$
Either, $f(0)=0$, or $f(0)=1$ or, $f(0)=-1$
Now, if $f(0)=0$ then by $p(x,0)$ we get,
$f(x)=f(x)^3$
$\rightarrow f(x)=0,1,-1$ [But the latter two are not acceptable]

Again if $f(0)-1$ then by $p(x,0)$ we get,
$f(x)+f(x)^2=1+f(x)^3$
Solving for $f(x)$ we get,
$f(x)=1,-1$ but $f(x)=-1$ is not acceptable.
Similarly for $f(0)=-1$,
$f(x)=-1,1$ [where the 2nd is not acceptable]

$\therefore f(x)=-1,0,1 $
But for which $x$, $f(x)=0$ and for which $x$, $f(x)=\pm1$?
I think you showed that the range of the function is $\{-1,0,1\}$ but that doesn't mean $f(x)$ has the same value for every input.